LSAT and Law School Admissions Forum

Get expert LSAT preparation and law school admissions advice from PowerScore Test Preparation.

User avatar
 Dave Killoran
PowerScore Staff
  • PowerScore Staff
  • Posts: 5850
  • Joined: Mar 25, 2011
|
#42033
Complete Question Explanation
(The complete setup for this game can be found here: lsat/viewtopic.php?t=15987)

The correct answer choice is (C)

This question is quite easy. From the Not Laws, we know that H cannot be placed first or seventh, and this information eliminates answer choices (A), (B), and (E). Next, the local condition in the question stem establishes that Z is in the fifth position. From the fourth rule, if Z is in the fifth position, them M must be in the sixth position. This information eliminates answer choice (D), and thus answer choice (C) is correct.

As you review the game, closely examine questions #2 and #4 as they both can be effectively attacked by using hypotheticals.
 kenswil
  • Posts: 2
  • Joined: Oct 25, 2013
|
#12123
I looked it up and the answer is C but if you put H in 4 then O must be in at least 7 which is two spaces. So if X is in 1 then V must be in 4, 2 then 5 and so on. I don't see how H can possibly be in 4. I'm sure I'm missing something foolish.
User avatar
 Dave Killoran
PowerScore Staff
  • PowerScore Staff
  • Posts: 5850
  • Joined: Mar 25, 2011
|
#12124
Hi Ken,

Thanks for the question. You're close, but it looks like you've assumed O has to be 7th, when it could be 8th.

If O was in 8, instead of being in 7, that would put H in 4 and O in 8, meaning there are 3 spaces between them. V could be in 7, and then X could be in 3 (satisfying X > V). P and T are 1 and 2 in some order.

The resulting hypotheticals are:

  • ..... ..... ..... P-T-X-H-Z-M-V-O

    ..... ..... ..... T-P-X-H-Z-M-V-O
In both cases, H can successfully be 4th.

Please let me know if that helps. Thanks!

Get the most out of your LSAT Prep Plus subscription.

Analyze and track your performance with our Testing and Analytics Package.